Đến nội dung

Nguyen Minh Hai nội dung

Có 652 mục bởi Nguyen Minh Hai (Tìm giới hạn từ 24-04-2020)



Sắp theo                Sắp xếp  

#640894 VMF's Marathon Bất Đẳng Thức Olympic

Đã gửi bởi Nguyen Minh Hai on 17-06-2016 - 17:32 trong Bất đẳng thức và cực trị

Vế phải là mũ $3$ mới phải  :mellow:

Mình nhầm :3 

 

P/s: Long Vá xóa bài giùm anh  :wacko:




#640882 VMF's Marathon Bất Đẳng Thức Olympic

Đã gửi bởi Nguyen Minh Hai on 17-06-2016 - 16:24 trong Bất đẳng thức và cực trị

Bài 35:(VQBC) Cho $a,b,c$ là các số thực ko âm, chứng minh rằng : 

$$2(\frac{a}b{+\frac{b}c{+\frac{c}{a}}})+1\geq \frac{21(a^{2}+b^{2}+c^{2})}{(a+b+c)^{2}}$$

 

Lời giải bài 35. (Cách khác)

 

Sử dụng bổ đề 

$$(x+y+z)^3 \geqslant \frac{27}{4}(a^2b+b^2c+c^2a+abc)$$

ta có

$$left(\frac{a}{b}+\frac{b}{c}+\frac{c}{a} \right )^3 \geqslant \frac{27}{4}\left( \frac{a^3+b^3+c^3}{abc}+1 \right)$$

 

Do đó BĐT cần chứng minh trở thành 

 

$$54\left( \frac{a^3+b^3+c^3}{abc}+1 \right) \geqslant \left( \frac{21(a^2+b^2+c^2)}{(a+b+c)^2}-1\right)^2$$

 

$$\Leftrightarrow 54\left( \frac{a^3+b^3+c^3}{abc}-3 \right) \geqslant \left( \frac{441(a^2+b^2+c^2)^2}{(a+b+c)^4}-49 \right)-\left(\frac{42(a^2+b^2+c^2}{(a+b+c)^2}-14\right)$$

 

$$\Leftrightarrow \frac{54(a+b+c)(\sum a^2-\sum ab)}{abc} \geqslant \frac{14(\sum a^2-\sum ab)\left[21\sum a^2+7(a+b+c)^2\right]}{(a+b+c)^4}-\frac{28(\sum a^2-\sum ab)}{(a+b+c)^2}$$

 

$$\Leftghtarrow \left(\sum a^2-\sum ab \right)\left( \frac{27(a+b+c)}{abc}+\frac{14}{(a+b+c)^2}-\frac{147\sum a^2+49(a+b+c)^2}{(a+b+c)^4} \right)$$

 

Do đó ta chỉ cần chứng minh 

 

$$\frac{27(a+b+c)}{abc}+\frac{14}{(a+b+c)^2}-\frac{147\sum a^2+49(a+b+c)^2}{(a+b+c)^4} \geqslant 0$$

 

Chuẩn hóa $a+b+c=1$ thì ta cần chứng minh 

$$\frac{27}{abc}+14 \geqslant 147(a^2+b^2+c^2)+49$$

$$\Leftrightarrow 147abc(a^2+b^2+c^2)+35abc \leqslant 27$$

 

BĐT cuối luôn đúng do 

$$abc \leqslant \frac{1}{27}$$

và 

$$a^2+b^2+c^2 \leqslant (a+b+c)^2 =1$$

 

BĐT được chứng minh. Xảy ra đẳng thức khi $a=b=c$




#639955 VMF's Marathon Bất Đẳng Thức Olympic

Đã gửi bởi Nguyen Minh Hai on 13-06-2016 - 00:20 trong Bất đẳng thức và cực trị

 

Bài toán 31 (Võ Quốc Bá Cẩn). Cho các số thực dương $a,b,c$ thỏa mãn $a+b+c=5$. Chứng minh
$$a^2b+c^2a+2abc \leq 20.$$

 

Lời giải bài 31. Đặt $f(a,b,c)=a^2b+c^2a+2abc$

Ta xét $2$ trường hợp.

Nếu $a \geqslant c$, ta sẽ chứng minh 

$$f(a,b,c) \leqslant f(a,b,a)$$

$$\Leftrightarrow (a-c)(a+c+2b) \geqslant 0$$

 

Nếu $a \leqslant c$, ta sẽ chứng minh 

$$f(a,b,c) \leqslant f(c,b,c)$$

$$\Leftrightarrow b(c^2-a^2)+c^2(c-a)+2bc(c-a) \geqslant 0$$

 

Do đó ta chỉ cần chứng minh BĐT trong trường hợp $a=c=\frac{5-b}{2}$

$$\Leftrightarrow \left ( \frac{5-b}{2}\right )^2b+\left ( \frac{5-b}{2} \right)^3+2b\left (\frac{5-b}{2} \right )^2 \leqslant 20$$

$$\Leftrightarrow  b^3-9b^2+15b-7 \leqslant 0$$

$$\Leftrightarrow (b-7)(b-1)^{2} \leqslant 0$$

BĐT cuối luôn đúng do $b<5$

Bài toán được chứng minh. Xảy ra đẳng thức khi $a=c=2,b=1$

 

Bài toán 32.  (Vasile Cirtoaje) Cho các số thực dương $a,b,c,d$. Chứng minh rằng 

$$2(a^3+1)(b^3+1)(c^3+1)(d^3+1) \geqslant (1+abcd)(1+a^2)(1+b^2)(1+c^2)(1+d^2)$$




#633449 Inequalities From 2016 Mathematical Olympiads

Đã gửi bởi Nguyen Minh Hai on 16-05-2016 - 14:38 trong Bất đẳng thức - Cực trị

 

Mình tổng hợp lại một số bài chưa có lời giải, mọi người cùng suy nghĩ thử nhé :)

 

Bài 28 (Russia). Cho bốn số thực dương $a, b, c, d$ thỏa mãn điều kiện $a+b+c+d=3.$ Chứng minh rằng $$\frac{1}{a^2}+\frac{1}{b^2}+\frac{1}{c^2}+\frac{1}{d^2}\le\frac{1}{a^2b^2c^2d^2},$$


$$\frac{1}{a^3}+\frac{1}{b^3}+\frac{1}{c^3}+\frac{1}{d^3}\le\frac{1}{a^3b^3c^3d^3}.$$

 

 

Lời giải 

 $(a)$       Giả sử $a \geqslant b \geqslant c \geqslant d$, BĐT cần chứng minh tương đương với

$a^{2}b^{2}c^{2}+a^{2}b^{2}d^{2}+a^{2}c^{2}d^{2}+b^{2}c^{2}d^{2} \leqslant 1$

Ta đưa BĐT về dạng đồng bậc

$a^{2}b^{2}c^{2}+a^{2}b^{2}d^{2}+a^{2}c^{2}d^{2}+b^{2}c^{2}d^{2} \leqslant \frac{1}{3^6}(a+b+c)^6$

Do $a \geqslant b \geqslant c \geqslant d$ nên ta

\[\begin{aligned} a^{2}b^{2}c^{2}+a^{2}b^{2}d^{2}+a^{2}c^{2}d^{2}+b^{2}c^{2}d^{2} &\leqslant a^{2}b^{2}c^{2}+a^{2}b^{2}d^{2}+a^{2}b^{2}cd+a^{2}b^{2}cd \\ &= a^{2}b^{2}(c+d)^{2} \\ &\leqslant \frac{1}{3^{6}}(a+b+c+d)^{6} \\ &=1\end{aligned}\]

Bài toán được chứng minh. $\square$ 

 $(b)$       Giả sử $a \geqslant b \geqslant c \geqslant d$, BĐT cần chứng minh tương đương với

$a^{3}b^{3}c^{3}+a^{3}b^{3}d^{3}+a^{3}c^{3}d^{3}+b^{3}c^{3}d^{3} \leqslant 1$

Ta đưa BĐT về dạng đồng bậc

$a^{3}b^{3}c^{3}+a^{3}b^{3}d^{3}+a^{3}c^{3}d^{3}+b^{3}c^{3}d^{3} \leqslant \frac{1}{3^9}(a+b+c)^9$

Theo BĐT $AM - GM$ ta

$ \frac{1}{3^9}(a+b+c+d)^9 \geqslant \frac{1}{3^9}\left(3\sqrt[3]{ab(c+d)} \right)^9 = a^3b^3(c+d)^3 $

Do đó ta chỉ cần chứng minh $a^{3}b^{3}c^{3}+a^{3}b^{3}d^{3}+a^{3}c^{3}d^{3}+b^{3}c^{3}d^{3} \leqslant a^3b^3(c+d)^3$ $\Leftrightarrow c^3d^3(a^3+b^3) \leqslant 3a^3b^3cd(c+d)$ $\Leftrightarrow c^2d^2(a^3+b^3) \leqslant 3a^3b^3(c+d)$

BĐT này luôn đúng do $a \geqslant b \geqslant c \geqslant d$

Bài toán được chứng minh. $\square$




#633413 Inequalities From 2016 Mathematical Olympiads

Đã gửi bởi Nguyen Minh Hai on 16-05-2016 - 01:25 trong Bất đẳng thức - Cực trị

 

Mình tổng hợp lại một số bài chưa có lời giải, mọi người cùng suy nghĩ thử nhé :)

 

 

Bài 27 (Iran Second Round). Cho ba số thực $0<a \leqslant b \leqslant c.$ Chứng minh rằng
$$\frac{(c-a)^2}{6c}\leq \frac{a+b+c}{3}-\frac{3}{\frac{1}{a}+\frac{1}{b}+\frac{1}{c}}.$$

 

Hình gửi kèm

  • iran second round.jpg



#633411 Inequalities From 2016 Mathematical Olympiads

Đã gửi bởi Nguyen Minh Hai on 16-05-2016 - 00:44 trong Bất đẳng thức - Cực trị

Một cách tiếp cận khác cho Bài 30

Bài 30 (EMMO). Cho ba số thực $a,b,c$ thỏa mãn điều kiện $a^2+b^2+c^2=4+abc.$ Chứng minh rằng $$(a^2+2)(b^2+2)(c^2+2)\ge 9+6(ab+bc+ca).$$
 

Đặt $t = \sqrt[3]{abc}$ thì từ giả thiết bài toán ta

$4+abc = a^{2}+b^{2}+c^{2} \geqslant 3\sqrt[3]{a^{2}b^{2}c^{2}}$

$\Leftrightarrow 4+t^3 \geqslant 3t^2  \Leftrightarrow  (t+1)(t+2)^2 \geqslant 0$

 

Từ đó suy ra $abc \geqslant -1$, kết hợp với giả thiết suy ra $a^{2}+b^{2}+c^{2} \geqslant 3$.

 

Ta BĐT quen thuộc sau $(a^{2}+2)(b^{2}+2)(c^{2}+2) \geqslant 3(a+b+c)^{2}   (1)$

 

Thật vậy, theo BĐT $Cauchy - Schwarz$ thì ta

 

$$(a+b+c)^{2} \leqslant (a^{2}+2)\left[1+\frac{(b+c)^2}{2}\right]$$

 

Do đó để chứng minh $(1)$ ta chỉ cần chứng minh

$$(b^{2}+2)(c^{2}+2) \geqslant 3\left[1+\frac{(b+c)^2}{2}\right]$$

 

$$\Leftrightarrow \frac{1}{2}(b-c)^2+(bc-1)^2 \geqslant 0$$

 

Do đó $(1)$ được chứng minh.

 

 Áp dụng $(1)$ thì ta quy bài toán về chứng minh

 

$$(a+b+c)^{2} \geqslant 3+2(ab+bc+ca)$$

 

$$\Leftrightarrow a^{2}+b^{2}+c^{2} \geqslant 3$$ 

 

BĐT này luôn đúng. Do đó bài toán được chứng minh 

 Xảy ra đẳng thức khi $a=b=c=-1$. $\square$




#633352 Inequalities From 2016 Mathematical Olympiads

Đã gửi bởi Nguyen Minh Hai on 15-05-2016 - 21:23 trong Bất đẳng thức - Cực trị

 

Mình tổng hợp lại một số bài chưa có lời giải, mọi người cùng suy nghĩ thử nhé :)

 

 

Bài 23 (Romania JBMO TST 2016). Với $m,n$ là hai số tự nhiên và ba số thực $x,y,z$ thuộc $[0,1].$ Chứng minh rằng

\[0 \leqslant x^{m+n}+y^{m+n}+z^{m+n}-x^my^n-y^mz^n-z^mx^n \leqslant 1.\]
Đẳng thức xảy ra khi nào ?

 

$(a)$      Chứng minh $ x^{m+n}+y^{m+n}+z^{m+n} \geqslant x^my^n+y^mz^n+z^mx^n$

Gọi $(a,b,c)$ hoán vị của $(x,y,z)$ sao cho $a \geqslant b \geqslant c$

 

Khi đó ta $a^m \geqslant b^m \geqslant c^m$ $a^n \geqslant b^n \geqslant c^n$.

 

Do đó theo BĐT Hoán vị ta

$$a^ma^n+b^mb^n+c^mc^n \geqslant a^mb^n+b^mc^n+c^ma^n$$

$$a^ma^n+b^mb^n+c^mc^n \geqslant a^mc^n+b^ma^n+c^ma^n$$

ta

$$a^ma^n+b^mb^n+c^mc^n=x^{m+n}+y^{m+n}+z^{m+n}$$

$$x^my^n+y^mz^n+z^mx^n \in \lbrace a^mb^n+b^mc^n+c^ma^n , a^mc^n+b^ma^n+c^ma^n \rbrace$$

Do đó vế trái của BĐT được chứng minh. Xảy ra đẳng thức khi $x=y=z$

$(b)$         Chứng minh

$$x^my^n+y^mz^n+z^mx^n+1 \geqslant x^{m+n}+y^{m+n}+z^{m+n}$$

 

Giả sử $x=max\lbrace x,y,z \rbrace$ thì BĐT được viết lại thành

 

$$(1-x^{m+n})+y^n(x^m-y^m)+z^m(x^n-z^n)+y^mz^n \geqslant 0$$

 

BĐT cuối luôn đúng do $x=max\lbrace x,y,z \rbrace$ $x,y,z \in [0;1]$.

 

Vậy vế phải BĐT được chứng minh. Xảy ra đẳng thức khi $x=1,y=z=0$ các hoán vị. $\square$




#631722 ĐỀ THI OLYMPIC CHUYÊN KHOA HỌC TỰ NHIÊN 2016

Đã gửi bởi Nguyen Minh Hai on 07-05-2016 - 12:05 trong Thi HSG cấp Tỉnh, Thành phố. Olympic 30-4. Đề thi và kiểm tra đội tuyển các cấp.

 

ĐỀ THI OLYMPIC CHUYÊN KHOA HỌC TỰ NHIÊN 2016

Ngày 1 (07/05/2016)
Câu 1. Giải hệ phương trình $$\begin{cases}x + y + xy = 3 \\ y^{3} + 13y = 6x^{2} + 8\end{cases}$$
Câu 2. Tìm tất cả các bộ số nguyên dương $(x, y, z)$ thỏa mãn phương trình $$7^{x} + 3^{y} = 2^{z}$$
Câu 3. Cho tam giác $ABC$ nhọn có tâm nội tiếp $I$. Đường thẳng qua $I$ vuông góc với $AI$ cắt cạnh $CA, AB$ lần lượt tại $M, N$. Gọi $E$ đối xứng $C$ qua $M$, $F$ đối xứng $B$ qua $N$. Giả sử $E, F$ đều lần lượt thuộc các đoạn thẳng $CA, AB$. Gọi $(K), (L)$ lần lượt là đường tròn ngoại tiếp các tam giác $ICE, IBF$.

  • Chứng minh rằng $(K)$ và $(L)$ tiếp xúc nhau tại $I$.
  • Gọi $EF$ theo thứ tự cắt $(K), (L)$ tại $P, Q$ khác $E, F$. Chứng minh rằng $EP = FQ$.

Câu 4. Cho dãy số $(a_{n})_{n\in\mathbb{Z}^{+}}$ xác định như sau $$\begin{cases}a_{1} = 0 \\ a_{2} = 1 \\ a_{2n} = 2a_{n} + 1 \\ a_{2n + 1} = 2a_{n}\end{cases}$$ với mọi $n \in \mathbb{Z}^{+}$. Chứng minh rằng tồn tại vô số số nguyên dương $k$ sao cho $a_{k} = 2016$ và tìm số nguyên dương $k$ nhỏ nhất thỏa mãn điều này.






Nguồn

 

 

Tóm tắt 

Câu 1 :

Phương trình $(2)$ tương đương với 

$6(x-1)(x+1)=(y-1)(y^2+y+14)$

Rút $y=\frac{3-x}{x+1}$ từ $(1)$ thay vào phương trình trên ta được 

$$(\frac{2(x-1)}{x+1}((x+1)^{2}+(\frac{3-x}{x+1})^2+\frac{3-x}{x+1}+14)=0$$

 

Câu 2 : 

Dễ chứng minh được $z$ chẵn, $x,y$ khác tính chẵn lẽ

Xét 2 trường hợp $x$ chẵn , $y$ lẻ và $x$ lẻ, $y$ chẵn

Cả 2 trường hợp đều đưa về dạng 

$7^y=(2^{z'}-3^{x'})(2^{z'}+3^{x'})$ ... 

Bộ $(x,y,z)$ thỏa mãn là $(1;2;4)$

 

Câu 4 : Bằng quy nạp dễ dàng chứng minh được

 - $a_{2^n}=2^n-1$ 

 - $a_{2^n+1}=2^{n}-2 $

...

- $a_{2^n+k}=2^{n}-k-1$ với $k$ là số nguyên dương sao cho $2^n+k<2^{n+1}$

...

Từ đó dễ dàng suy ra kết quả bài toán.




#624063 Hỏi có nhiều nhất bao nhiêu tam giác không đồng dạng với nhau.

Đã gửi bởi Nguyen Minh Hai on 01-04-2016 - 20:09 trong Tổ hợp và rời rạc

Một n-giác đều được chia thành các tam giác bởi $n-3$ đường chéo không cắt nhau trừ tại đỉnh. Hỏi có nhiều nhất bao nhiêu tam giác không đồng dạng với nhau.




#623931 $\sum_{x \in X}x =n^k$

Đã gửi bởi Nguyen Minh Hai on 31-03-2016 - 21:59 trong Số học

Chứng minh rằng tồn tại tập hợp $A$ có $2016$ phần từ là các số nguyên dương sao cho với mọi tập $X\subset A, X \neq \varnothing$ thì ta có $\sum_{x \in X} x$ là lũy thừa với số mũ nguyên lớn hơn $1$ của một số nguyên dương nào đó.




#621233 Tìm tất cả cặp số nguyên dương (a;b) sao cho $ \dfrac{a^b+b...

Đã gửi bởi Nguyen Minh Hai on 19-03-2016 - 20:43 trong Số học

Em không nhớ rõ nhưng hình như bài mà bác nói cái tử số là $a^2b+b$ chứ không phải là $a^b+b$. 

 
 

Bài này có lẽ khó hơn bài $Rio plate 2002$ rất nhiều. Hi vọng có lời giải bài này sớm :)

Không nhầm đâu bạn. Bày này chính minh đăng trên AoPs vài tháng trước và sau đó có ai đó đăng lại trên VMF. Nếu như sách viết không nhầm thì bài này chính là Rio Plate 2002. 




#621153 Tìm tất cả cặp số nguyên dương (a;b) sao cho $ \dfrac{a^b+b...

Đã gửi bởi Nguyen Minh Hai on 19-03-2016 - 15:15 trong Số học

Bài đó không quá khó, tuy nhiên bài này có lẽ chế từ bài đó nhưng khó hơn.

Hai bài y chang nhau mà 




#621092 Tìm tất cả cặp số nguyên dương (a;b) sao cho $ \dfrac{a^b+b...

Đã gửi bởi Nguyen Minh Hai on 19-03-2016 - 01:03 trong Số học

Bài này có trong sách của nhóm tác giả Nguyễn Văn Mậu, Đặng Hùng Thắng, Trần Nam Dũng và được giới thiệu là đề thi Rio Plate 2002 tuy nhiên không có lời giải.




#611003 Chứng minh rằng với mọi số nguyên dương $n$ đều tồn tại một hình vô...

Đã gửi bởi Nguyen Minh Hai on 25-01-2016 - 21:46 trong Tổ hợp và rời rạc

Như thế anh cũng không thấy ổn lắm.

Vì mọi hình vuông $n \times n$ đều là không 'tốt', tức là luôn tồn tại đoạn thẳng nối từ $O$ tới một điểm nguyên trong hình vuông mà không chứa điểm nguyên khác. 

Hình vuông "không tốt" em nghĩ là mọi đoạn thẳng nối tâm $O$ tới một điểm nguyên trên cạnh hoặc trong hình vuông đều không chưa điểm nguyên khác.




#610657 ính tổng tất cả các số "tốt" không vượt quá 2016

Đã gửi bởi Nguyen Minh Hai on 24-01-2016 - 00:46 trong Tổ hợp và rời rạc

Ta gọi một số nguyên dương là "tốt" nếu tổng các ước số của số đó là số chính phương. Tính tổng tất cả các số "tốt" không vượt quá 2016

 




#610476 Chứng minh rằng với mọi số nguyên dương $n$ đều tồn tại một hình vô...

Đã gửi bởi Nguyen Minh Hai on 23-01-2016 - 00:50 trong Tổ hợp và rời rạc

Mình nghĩ bài toán có gì đó không đúng. Chẳng hạn với một hình vuông $n \times n$ bất kì, lấy đoạn thẳng từ $O$ tới điểm nguyên gần $O$ nhất thì đoạn thẳng đoạn thẳng đó đâu có chứa điểm nguyên nào khác hai đầu mút.

À em gõ đề nhầm :3 

Chứng minh với mọi số nguyên dương $n$ thì tồn tại một hình vuông $n.n$ không "tốt" 




#609809 $\frac{a^{2}+b^{2}+c^{2}}...

Đã gửi bởi Nguyen Minh Hai on 19-01-2016 - 15:11 trong Bất đẳng thức - Cực trị

Cho a,b,c dương. Chứng minh

$\frac{a^{2}+b^{2}+c^{2}}{ab+bc+ca}+\frac{1}{3}\geq \frac{8}{9}\sum \frac{a}{b+c}$

BĐT $\Leftrightarrow \frac{a^2+b^2+c^2}{ab+bc+ca}-1 \geqslant \frac{8}{9}\left ( \sum \frac{a}{b+c}-\frac{3}{2} \right )$

 

        $\Leftrightarrow \sum \frac{(a-b)^2}{2(ab+bc+ca)} \geqslant \frac{8}{9}\sum \frac{(a-b)^2}{2(a+c)(b+c)}$

 

        $\Leftrightarrow (a-b)^2S_c+(b-c)^2S_a+(c-a)^2S_b \geqslant 0$

 

trong đó $S_c=\frac{1}{2(ab+bc+ca)}-\frac{4}{9(a+c)(b+c)}, S_b, S_a$ xác định tương tự và dễ dàng chứng minh được $S_a,S_b,S_c \geqslant 0$




#608877 Chứng minh rằng với mọi số nguyên dương $n$ đều tồn tại một hình vô...

Đã gửi bởi Nguyen Minh Hai on 13-01-2016 - 23:21 trong Tổ hợp và rời rạc

Ta định nghĩa hình vuông "tốt" là một hình vuông có $4$ đỉnh là các điểm nguyên, đồng thời các đoạn thẳng nối tâm $O$ với tất cả các điểm nguyên trên biên và trong hình vuông chứa ít nhất một điểm nguyên khác hai đầu mút. Chứng minh rằng với mọi số nguyên dương $n$ đều tồn tại một hình vông tốt dạng $n.n$




#608215 Chứng minh rằng dãy $(p_n)$ bị chặn trên.

Đã gửi bởi Nguyen Minh Hai on 09-01-2016 - 21:06 trong Số học

Cho dãy $(p_n)$ là dãy các số nguyên tố thỏa mãn với mọi $n \geqslant 3$ thì ta có $p_n$ là ước nguyên tố lớn nhất của $p_{n-1}+p_{n-2}+2000.$.

Chứng minh rằng dãy $(p_n)$ bị chặn trên. 




#607857 Chứng minh rằng có ít nhất một phần tử của $A$ có thể biểu diễn như...

Đã gửi bởi Nguyen Minh Hai on 07-01-2016 - 23:48 trong Tổ hợp và rời rạc

Cho $k,n \geqslant 1$ là các số tự nhiên và $A$ là tập hợp gồm $(k-1)n+1$ số nguyên dương, mỗi số này đều không vượt quá $kn$. Chứng minh rằng có ít nhất một phần tử của $A$ có thể biểu diễn như tổng của $k$ phần từ trong $A$. ($k$ phần tử này không nhất thiết phải khác nhau)




#607814 Chứng minh rằng với mọi số nguyên dương $n$, tồn tại hữu hạn phép b...

Đã gửi bởi Nguyen Minh Hai on 07-01-2016 - 20:57 trong Tổ hợp và rời rạc

Từ một số nguyên dương $n$, ta tác động hai phép toán sau đây : 

   1. Nhân nó với một số nguyên dương tùy ý.

   2. Xóa bỏ các chữ số $0$ trong biểu diễn thập phân của nó.

 

Chứng minh rằng với mọi số nguyên dương $n$, tồn tại hữu hạn phép biến đổi ta có thể đưa $n$ thành số $9$. 




#603882 TOPIC ôn luyện VMO 2016

Đã gửi bởi Nguyen Minh Hai on 18-12-2015 - 22:38 trong Thi HSG Quốc gia và Quốc tế

Bài 14: (Số học) Cho $a,b \in \mathbb{Z}$ sao cho $|a|,|b|>1$ và tập hợp $S=\left \{ n \in \mathbb{N}^* \mid \exists m \in \mathbb{N}^*,a^m+b \vdots a^n+1 \right \}$ có vô hạn phần tử. Chứng minh rằng: 

$\exists k \in \mathbb{N^*}, |b|=|a|^k$




#600369 ĐỀ KIỂM TRA TOÁN 10 TRƯỜNG THPT CHUYÊN QUỐC HỌC 2015 - 2016

Đã gửi bởi Nguyen Minh Hai on 27-11-2015 - 22:07 trong Thi HSG cấp Tỉnh, Thành phố. Olympic 30-4. Đề thi và kiểm tra đội tuyển các cấp.

Mới làm được câu a):

Có $x$ tập con có 2 phần tử nên tổng $S$ bằng tổng của $2x$ phần tử.

Vì các phần tử phân biệt nên $S\geq 1+2+...+2x=x(2x+1)$

Theo giả thiết thì kích thước của các tập con trong $x$ tập con đó phân biệt từng đôi một và không vượt quá $2016$.

Do đó: $S\leq (2017-1)+(2017-2)+...+(2017-x)=\frac{-x^2+4033x}{2}$

=> ĐPCM 

Câu b sử dụng câu a suy ra được $x \leqslant 806$. Chỉ cần chỉ ra $806$ tập thõa mãn là được :3 




#600308 ĐỀ KIỂM TRA TOÁN 10 TRƯỜNG THPT CHUYÊN QUỐC HỌC 2015 - 2016

Đã gửi bởi Nguyen Minh Hai on 27-11-2015 - 18:26 trong Thi HSG cấp Tỉnh, Thành phố. Olympic 30-4. Đề thi và kiểm tra đội tuyển các cấp.

attachicon.gifUntitled.png

a) Theo một bổ đề quen thuộc thì $\widehat{AIB}=90$ độ.

Sau đó như thế nào thì dễ rồi.

b) 

Gọi giao điểm của $MI$ với $PQ$ là $N$, $K,L$ theo thứ tự là giao của $PQ$ với $AX,BX$. Các điểm còn lại như hình vẽ.

Dễ thấy điểm $I$ cố định và $EG=DJ$

$PQ$ là trục đẳng phương nên dễ chứng minh $PQ//ED$.

Lại có $IN//DL$ nên $INLD$ là hình bình hành dẫn đến $IN=DL=\frac{DJ}{2}$

Mà $DJ=DC+CL=DC+CO=BC-BR+AC-AO=AC+BC-BR-(AE+EG)=AC+BC-AB-DJ$.

Nên $DJ$ có độ dài không đổi dẫn đến $IN$ có độ dài không đổi.

Mà $I$ cố định và $IN//BC$ nên $N$ cố định.

=> ĐPCM

Ngồi thi còn ko vẽ được cái hình nữa :'(




#600294 ĐỀ KIỂM TRA TOÁN 10 TRƯỜNG THPT CHUYÊN QUỐC HỌC 2015 - 2016

Đã gửi bởi Nguyen Minh Hai on 27-11-2015 - 17:03 trong Thi HSG cấp Tỉnh, Thành phố. Olympic 30-4. Đề thi và kiểm tra đội tuyển các cấp.

TRƯỜNG THPT CHUYÊN QUỐC HỌC - HUẾ                                         ĐỀ KIỂM TRA TOÁN 10

                            TỔ TOÁN                                                                                          Thời gian : 120 phút.

 

Câu 1: (4điểm) Cho $a,b,c$ là các số thực dương thỏa mãn $a+b+c=1$. Chứng minh:

                      $\frac{a^2+b^2}{c}+\frac{b^2+c^2}{a}+\frac{c^2+a^2}{b} \geqslant 2$

 

Câu 2: (6đ) Cho tam giác $ABC$, điểm $X$ thay đổi trên đường thẳng $BC$ sao cho $C$ nằm giữa $B$ và $X$. Gọi $M$ là trung điểm của $AB$. Đường tròn nội tiếp tam giác $ABX$ tiếp xúc với $BX, AX$ lần lượt tại $D,E$.

     a) Gọi $I$ là giao điểm của $DE$ và phân giác trong của góc $B$. Chứng minh $MI || BC$

     b) Các đường tròn nội tiếp các tam giác $ABX$ và $ACX$ cắt nhau tại $P$ và $Q$. Chứng minh rằng đường thẳng $PQ$ đi qua một điểm cố định.

 

Câu 3: (5đ) Phân chia tập $A=\left \{ 1;2;3;...;2016 \right \}$ thành $1008$ tập con rời nhau, mỗi tập gồm hai phần tử, ta gọi kích thước của mỗi tập con là tổng hai phần tử của nó. Gọi $x$ là số các tập con mà kích thước của chúng phân biệt từng đôi một và không vượt quá $2016$, gọi $S$ là tổng tất cả các phần tử của các tập con đó.

     a) Chứng minh rằng      $x(2x+1) \leqslant S \leqslant \frac{-x^2+4033x}{2}$

     b) Tìm giá trị lớn nhất của $x$.

 

Câu 4: (5đ) Với một số nguyên dương $n$ cho trước, gọi $f(n)$ là số nguyên dương nhỏ nhất sao cho $\sum_{k=1}^{f(n)} k$  là bội của $n$. Chứng minh rằng $f(n)=2n-1$ khi chỉ khi $n$ là lũy thừa của $2$

 

Spoiler